what is the degree of the polynomial. 5x²+8x²- 4x⁴+x. (a) 2 (b) 1 (c) 4 (d) 8​

Answers

Answer 1

Answer:

4

Step-by-step explanation:

5x²+8x²- 4x⁴+x

The degree is the highest power

The highest power is 4

The degree is 4


Related Questions

The histogram represents the distributions of essay scores for high school sophomores and juniors in a contest. Which statements are true about the data used to create the histogram? Select three options.

Answers

Question options:

A. The mean is the best comparison of the measures of center.

B. The juniors tended to have higher essay scores than the sophomores.

C. The medians of both data sets are equal.

D. The interquartile range is the best comparison of the measure of variability.

E. A histogram is the best way to show that both distributions are nearly symmetric.

Answer:

B. The juniors tended to have higher essay scores than the sophomores.

C. The medians of both data sets are equal.

E. A histogram is the best way to show that both distributions are nearly symmetric.

Explanation:

A histogram is used to represent data in a bar graph like manner. In a histogram, the bars lie along the x axis, each within the range of intervals while the y axis is used to count the frequency of occurrence that determines the area of each bar in the graph. Using a histogram, we can find B, C, and E from the options given above.

Determine whether the given triangles are congruent

Answers

Answer:

Yes they are congruent and it is explained below

Step-by-step explanation:

There are 5 postulates when testing for congruency and they are;

SSS, SAS, ASA, AAS and HL.

S represents side

A represents angle

H represents hypotenuse

L represents leg

Now, in this our question, we can see that the 3 angles and the three sides of both triangles are equal to each other.

Thus, we can make use of SSS postulate whereby we say the 3 sides of the first triangle are equal to the 3 sides of the second triangle.

Thus, they are congruent.

can I please can some help on this question ​

Answers

Answer: The answer be 87 for both answer's

Step-by-step explanation:

because, {B-3=90} and, 0.< a < 90. and I calculated and got 87 for both questions ( I hope it correct!)

which of the following numbers must be added to complete the square in the equation below x^2+16x=5​

Answers

Answer:

8

Step-by-step explanation:

I was stumped on this question too so I figured I might as well learn how to do it. All you have to do is divide the middle number (16x) by 2. Pretty easy! Hope this helps anyone who needs it!

A 2km long stretch of road needs to be resurfaced. The width of the road is 8.3m. Model this stretch of road as a rectangle.Work out the area of road that needs to be resurfaced in metre squared. Resurfacing this road costs £30 per square metre What is the total cost to resurface this road?

Answers

1 km = 1000 meters

2 km = 2 x 1000 = 2,000 meters

Area = length x width

Area = 2,000 x 8.3 = 16,600 square meters

Cost:

Multiply total area by cost per square meter:

16,600 x 30 = £498,000

A newsletter publisher believes that 30% of their readers own a Rolls Royce. A testing firm believes this is inaccurate and performs a test to dispute the publisher's claim. After performing a test at the 0.02 level of significance, the testing firm fails to reject the null hypothesis. What is the conclusion regarding the publisher's claim

Answers

Answer:

The evidence is not sufficient at the 0.02 level of significance to reject the claim that the percentage is 30%.

Step-by-step explanation:

Let's first state the null and alternative hypotheses for this research;

H0: p = 0.30

Ha: p ≠ 0.30

Now, since the bull hypothesis is the claim and he fails to reject the null hypothesis from the research, we will conclude that;

The evidence is not sufficient at the 0.02 level of significance to reject the claim that the percentage is 30%.

Which expression is equivalent to this quotient?

[tex]\frac{1}{x+5} \\\frac{x+3}{5x+15}[/tex]

Answers

I think this was your problem.. hope this helps!

Jason and Donny painted a house and received $1.200. To complete the painting job Jason painted 4 hours 25 minutes and Donny spent 2 hours and 15 minutes. If they split the $1.200 in proportion to the amount of time each spent painting, how much did Donny receive?​

Answers

Answer:

$405

Step-by-step explanation:

Total hours = 4 hrs 25 min + 2 hrs 15 min = 6 hrs 40 min = 400 min

4 hrs 25 min = 265 minutes

2 hrs 15 min = 135 min

Donny's share = [tex]\frac{135}{400} of 1200[/tex]

                        [tex]=\frac{135}{400}*1200=135*3\\\\= 405[/tex]

On the graph shown, what is f(-2)

Answers

Answer:

3 because when x=2 the lines are at y 1 and 3, but the y 1 isn't shaded, so the answer is 3

Please help me the picture is above I’ll mark as brainliest. I really need the answer

Answers

I think that is is 65 cubic inches in a box

Help now now the other 2 are Wednesday and Thursday

Answers

Wednesday
-4-9= -13
Wednesday had a temp of -13°c so its the correct answer

What is the DMS measure of an angle that is 3.7 radians?
A. 11° 37' 26"
B. 0° 3' 52.5"
c. 211° 59' 39.6"
D. 3° 42' 0"

Answers

C. Because it equals to the degrees to the 3.7 radians

In parallelogram LMNP if PQ=21 find QM.

Answers

Answer:

QM = 21

Step-by-step explanation:

The diagonals of a parallelogram bisect each other, then

QM = PQ = 21

Write an equation of the parabola that passes through the point (46, - 150) and has x-intercepts - 4 and 56 . Then find the average rate of change from x= -4 to x=6.

An equation is y =

The average rate of change is

Answers

Answer:

6w83ywytahausjsshshfwgwyee

Fbjdbdbdhhdhdbddbdbbdbddbbd so yea

Can someone check tell me why 1) is correct and why 2) is incorrect?!

Willing to give brainliest

Answers

Answer:

1) yes

2) no

Step-by-step explanation:

1)

[tex]\frac{1}{2} *\frac{2}{2}=\frac{2}{4}[/tex]

This is correct because 1 time 2 equals 2 and 2 times 2 equals 4.

2)

[tex]\frac{1}{2} * \frac{?}{?}= \frac{4}{6}[/tex]

[tex]\frac{1}{2} *\frac{4}{3} =\frac{4}{6}[/tex]

This is incorrect because the numerator multiplied is different from the denominator multiplied.

-------------------------------

[tex]\frac{1}{2}[/tex] =  [tex]\frac{2}{4}[/tex] = [tex]\frac{3}{6}[/tex] = [tex]\frac{4}{8}[/tex] = [tex]\frac{5}{10}[/tex] = [tex]\frac{6}{12}[/tex] ...

Sorry I didn't know how to explain that much but I hope you understand.

Arrange the following in ascending order. 1/2 1/4 3/4 1​

Answers

Answer:

1/4

1/2

3/4

1

Step-by-step explanation:

1/2 = 0.5

1/4 = 0.25

3/4 = 0.75

1 = 1

a tablet is on sale for 30% off. the regular price is $185. how much money will you save?

Answers

Answer:

185×30÷100=55.5

HERES YOUR answer

Over what interval is the function in this graph constant?

Answers

Answer:

[tex]\text{A. }1\leq x \leq6[/tex]

Step-by-step explanation:

A constant intervals occurs when the function is neither increasing or decreasing. In the graph, we see this occurs from [tex]x=1[/tex] to [tex]x=6[/tex]. Therefore, the interval in which the graph is constant is [tex]\boxed{\text{A. }1\leq x \leq6}[/tex]

Please show me 3+2 (2x - 4) and can you please show the work?

Answers

Answer:

4x -5

Step-by-step explanation:

3+2 (2x - 4)

Distribute

3 + 2*2x - 2*4

3+4x - 8

Combine like terms

4x +3-8

4x -5

Answer:

4x - 5

Step-by-step explanation:

3+ 2 (2x) + 2 (-4)

3+4x - 8

The weight Wkg of a metal bar varies jointly as it's length L and the square of it's diameter D mm. If W=140 when D =4 and L=54, find D interm of W and L

Answers

Answer:

D = [tex]\sqrt{\frac{216W}{35L} }[/tex]

Step-by-step explanation:

From the given question, the expression showing the relationship among the weight, length and diameter of the metal bar is;

W [tex]\alpha[/tex] L[tex]D^{2}[/tex]

W = kL[tex]D^{2}[/tex]

where k is the constant of proportionality.

When W = 140, D = 4 and L = 54, then;

140 = k(54)[tex](4)^{2}[/tex]

      = 864k

k = [tex]\frac{140}{864}[/tex]

  = [tex]\frac{35}{216}[/tex]

k = [tex]\frac{35}{216}[/tex]

⇒ W = [tex]\frac{35LD^{2} }{216}[/tex]

So that;

35L[tex]D^{2}[/tex] = 216W

[tex]D^{2}[/tex] = [tex]\frac{216W}{35L}[/tex]

D = [tex]\sqrt{\frac{216W}{35L} }[/tex]

what are 2 consecutive odd integers whose sum is 36.

Answers

Two consecutive odd integers with a sum of 36 are 17 and 19.

Hope it helps you I'm from ph❤️

Answer:

Two consecutive odd integers with a sum of 36 are 17 and 19.

Step-by-step explanation:

hope i helped

what is 2+a, when a is 17?

Answers

Answer:

19

Step-by-step explanation:

If a=17, then:

2 + 17 = 19

---------------------------------------------------------------------------------------------------------------

Have a great summer :)

the answer is 19 because
substitute a for 17
17+2= 19

A factory puts 2,000 picture frames in each shipment. How many picture frames will the
factory put in 2 shipments?
picture frames

Answers

4,000

I know this because 2,000 • 2 = 4,000

what is the value of w to the nearest degree

Answers

Answer hope it helps you

Which ratio is not equivalent to 3:6?
a. 1:2
b. 4:10
c. 10:20
d. 15:30

Answers

B. 4:10 is the answer that you are looking for

Answer:

B. 4:10

Step-by-step explanation:

A. 3:6 is equal to 1:2, which is our given answer for A.

B. Has no equivalent to 3:6

C. 10:20 can be simplified to 1:2 which 3:6 can also be simplified to… giving our answer.

Finally, D. You can multiply equally by 5 to get 15:30 and divide to get back to 3:6… it’s equal ;)

Hope this helps, good luck!

What is the Order of Magnitude of the number of pennies equivalent to a quarter?​

Answers

Answer:

It is mainly used while doing scientific notation. If two numbers differ by one order of magnitude, one is about ten times larger than the other. If they differ by two orders of magnitude, they differ by a factor of about 100.A quarter is 0.25 dollars so the ratio between 1 dollar and a quarter is 1/0.25(it means the dollar and the quarter which is 0.25 dollars) and that equals =4

what is the total cost for an item that costs $140.00 and has a sales tax rate of 7.5%

Answers

Answer:

$150.5

Step-by-step explanation:

CP of an item = $140

sales tax rate = 7.5%

sales tax amount = sales tax rate of CP

=7.5% of CP

= 7.5/100 * $140

=$1050/100

=$10.5

total cost with tax = CP + tax rate

=$140 + $10.5

=$150.5

A petrol can is a rectangular prism with base measurements 15 cm by 30 cm. If the can has capacity 18 liters, find its height.​

Answers

Answer:

40

Step-by-step explanation:

The first step is to calculate the base area of the prism

= 15×30

= 450

The volume is then calculated as follows

= 450×h

= 450h

The capacity is 18 liters

= 18×1000

= 18,000

Therefore the height of the rectangular prism can be calculated as follows

450h= 18,000

h= 18,000/450

= 40

Hence the height is 40 cm

Select all the numbers that are irrational.

Answers

9514 1404 393

Answer:

  π, √3, 3√2

Step-by-step explanation:

A number is irrational if it is not a decimal number of finite length or a repeating decimal number.

The irrational numbers in the group are shown below.

Help asappppp hbshhshshhshshsshshshs

Answers

9 on the top as well so 9 x 2 18 3x2 6 3

Other Questions
DO U KNOW CARRYMINATI please help :DDDDDDD If the amplitude of a sound wave is increased by a factor of four,how does the energy carry by the sound wave in each time interval change? How could you correctly combine the first three sentences of the passage below to add variety? Checksaw Amir. I was visiting New York. He lives there. We went to an Italian restaurant and ate big plates of spaghetti.was a fun outing.D saw Amir, who in lives in New York, when I was visiting.O When I was visiting New York, I saw Amir, who lives there.U When I was visiting New York; saw Amir, who lives there.0 saw Amir when I was visiting New York, which is where he lives. iSD I saw Amir, when I was visiting New York, that is where he livesANSWER IS A,B,D Help me pls this is hard PLZ HELPPP I need to pass this!! if a loaf of bread coast 1.52 how much change will you get from 10 if you buy 4loaves The Marked price of a jacket at a dealer's shop is 2400. The dealer allows a discount of 15% to the retailer. As the company's policies the marked price of the retailer should be the same as the dealer's marked price. What SP should the retailer fix for the jacket? What would its final price be if 10% VAT is imposed? What would be the actual gain of retailer?Ans 2280,2502,240 Study the table about Earths interior. James inherited 19.5hectares of land from his father. The area of the land he inherited is 65% of the total land his father owned. Calculate the total area of the land that his father owned The osmotic pressure of a solution formed by dissolving 0.231 g of the protein in 150 mL of water at 298 K is found to be 2.054 torr. Use the tax table to help answer the following question.A 9-column table with 7 rows is shown. Column 1 is labeled If the wages are at least with entries 720, 740, 760, 780, 800, 820, 840. Column 2 is labeled But less than with entries 740, 760, 780, 800, 820, 840, 860. Column 3 is labeled And the number of withholding allowances is 0, the amount of income tax withheld is, with entries 80, 83, 86, 89, 92, 95, 98. Column 4 is labeled And the number of withholding allowances is 1, the amount of income tax withheld is, with entries 62, 65, 68, 71, 74, 77, 80. Column 5 is labeled And the number of withholding allowances is 2, the amount of income tax withheld is, with entries 44, 47, 50, 53, 56, 59, 62. Column 6 is labeled And the number of withholding allowances is 3, the amount of income tax withheld is, with entries 26, 28, 31, 34, 37, 40, 43. Column 7 is labeled And the number of withholding allowances is 4, the amount of income tax withheld is, with entries 14, 16, 18, 20, 22, 24, 26. Column 8 is labeled And the number of withholding allowances is 5, the amount of income tax withheld is, with entries 1, 3, 5, 7, 9, 11, 13. Column 9 is labeled And the number of withholding allowances is 6, the amount of income tax withheld is, with entries 0, 0, 0, 0, 0, 0, 1.Luce is single and making $763 biweekly. She claims no federal withholding allowances. If the state tax is 19% of the federal tax, how much in state tax does Luce contribute? a.$12.92b.$15.13c.$15.77d.$16.34 write a paragraph explaing why collect a can is an example of sustainable development Help please Ill give you brainless what is the domain of the function represented by the graph Algebra 2 Problem! Rational Roots Read the passage from "The Tell-Tale Heart.I think it was his eye! yes, it was this! He had the eye of a vulturea pale blue eye, with a film over it. Whenever it fell upon me, my blood ran cold; and so by degreesvery graduallyI made up my mind to take the life of the old man, and thus rid myself of the eye forever. Now this is the point. You fancy me mad. Madmen know nothing. But you should have seen me. You should have seen how wisely I proceededwith what cautionwith what foresightwith what dissimulation I went to work! I was never kinder to the old man than during the whole week before I killed him.How is suspense created for the reader in this passage?The reader wonders where the old man lives.The reader is waiting to find out details of the murder.The reader envisions a vulture with pale blue eyes.The reader does not know why the old man is killed. Which 2 options are correct? what event happened immediately following the election of lincoln in 1860? which of the following items has the most inertia while at rest